Difference between revisions of "2024 AMC 12B Problems/Problem 23"
Username2333 (talk | contribs) |
|||
Line 13: | Line 13: | ||
Finally, using the pythagorean theorem, we can find that <math>{\overline{IV}}^2={\overline{AV}}^2-{\overline{IA}}^2= {((\sqrt{2}+2)/2)}^2 - (2+\sqrt{2})/2 = \boxed{(1+\sqrt{2})/2}</math> which is answer choice <math>\boxed{B}</math>. | Finally, using the pythagorean theorem, we can find that <math>{\overline{IV}}^2={\overline{AV}}^2-{\overline{IA}}^2= {((\sqrt{2}+2)/2)}^2 - (2+\sqrt{2})/2 = \boxed{(1+\sqrt{2})/2}</math> which is answer choice <math>\boxed{B}</math>. | ||
+ | |||
+ | ==Solution 2 (Less computation)== | ||
+ | |||
+ | Let <math>O</math> be the center of the regular octagon. Connect <math>AD</math>, and let <math>I</math> be the midpoint of line segment <math>AD</math>. It is easy to see that <math>VI=\frac{1}{2} AD=\frac{1+\sqrt{2}}{2}</math> and <math>OI=\frac{1}{2}AH=\frac{1}{4}</math>. Hence, | ||
+ | <cmath>VO^2=VI^2-OI^2</cmath> | ||
+ | <cmath>=\left(\frac{1+\sqrt{2}}{2}\right)^2-\frac{1}{4}</cmath> | ||
+ | <cmath>=\frac{1+\sqrt{2}}{2}</cmath> | ||
+ | Hence, the answer is <math>\boxed{B}</math>. |
Revision as of 04:06, 14 November 2024
Problem
A right pyramid has regular octagon with side length as its base and apex Segments and are perpendicular. What is the square of the height of the pyramid?
Solution 1
To find the height of the pyramid, we need the length from the center of the octagon (denote as ) to its vertices and the length of AV.
From symmetry, we know that , therefore is a 45-45-90 triangle. Denote as so that . Doing some geometry on the isosceles trapezoid (we know this from the fact that it is a regular octagon) reveals that and .
To find the length , we cut the octagon into 8 triangles, each witha smallest angle of 45 degrees. Using the law of cosines on we find that .
Finally, using the pythagorean theorem, we can find that which is answer choice .
Solution 2 (Less computation)
Let be the center of the regular octagon. Connect , and let be the midpoint of line segment . It is easy to see that and . Hence, Hence, the answer is .